스펙트럼 열: 두 판 사이의 차이

내용 삭제됨 내용 추가됨
편집 요약 없음
편집 요약 없음
34번째 줄:
스펙트럼 열이 퇴화하는 것은 스펙트럼 열이 수렴하는 것보다 더 강한 조건이다.
 
=== 5항제1 완전열사분면 스펙트럼 열 ===
'''제1 사분면 스펙트럼 열'''(第一四分面, {{llang|en|first-quadrant spectral sequence}})는 다음 조건을 만족시키는 스펙트럼 열이다.
==== 코호몰로지 5항 완전열 ====
* 만약 <math>p<0</math> 또는 <math>q<0</math>이라면 <math>E^{p,q}_r=0</math>
수렴하는 스펙트럼 열
즉, 모든 쪽에서 성분이 오직 제1 [[사분면]]에서만 [[영 대상]]이 아닌 스펙트럼 열이다. 사실, <math>r</math>번째 쪽에서 성분이 제1 사분면에만 존재한다면 그 다음에 오는 모든 쪽에서 성분들은 제1 사분면에서만 존재하게 된다. 따라서, 이 조건은 첫 번째 쪽에서만 확인하면 된다.
:<math>E^{p,q}_2\Rightarrow_p E_\infty^{p+q}</math>
이 주어졌다고 하고, <math>p<0</math> 또는 <math>q<0</math>이라면 그 성분이 자명하다고 하자. 그렇다면 다음과 같은 [[완전열]]이 존재한다.
:<math>0\to E_2^{1,0}\to E_\infty^1\to E_2^{0,1}\xrightarrow{d_2^{0,1}} E_2^{2,0}\to E^2_\infty</math>
이를 '''5항 [[완전열]]'''(五項完全列, {{llang|en|five-term exact sequence}})이라고 한다.
 
제1 사분면 스펙트럼 열은 항상 수렴한다. 구체적으로, 제1 사분면 코호몰로지 스펙트럼 열의 경우
이는 구체적으로 다음과 같다. 스펙트럼 열의 성분들이 오직 제1사분면에서만 자명하지 않으므로,
:<math>E^{p,q}_\infty=
E^{p,q}_{\max\{p+1,q+2\}}</math>
이며, 호몰로지 스펙트럼 열의 경우 역시
:<math>E_{p,q}^\infty=E_{p,q}^{\max\{p+1,q+2\}}</math>
이다.
 
==== 코호몰로지 5항제1 완전열사분면 스펙트럼 열 ====
제1 사분면 코호몰로지 스펙트럼 열
:<math>E^{p,q}_2\Rightarrow_p E_\infty^{p+q}</math>
이 주어졌다고 하자. 그렇다면, 셋째 쪽의 처음 몇 성분들은 다음과 같다.
:<math>\left|\underline{\begin{matrix}
\vdots&\vdots&\vdots&\vdots\\
E^{0,2}_3&E^{1,2}_3&E^{2,2}_3&E^{3,2}_3&\cdots\\
E^{0,1}_3&E^{1,1}_3&E^{2,1}_3&E^{3,1}_3&\cdots\\
E^{0,0}_3&E^{1,0}_3&E^{2,0}_3&E^{3,0}_3&\cdots
\end{matrix}}\right.
\qquad=\qquad\left|\underline{\begin{matrix}
\vdots&\vdots&\vdots&\vdots\\
\ker d_2^{0,2}&\ker d_2^{1,2}&\tfrac{\ker d_2^{2,2}}{\operatorname{im}d_2^{0,3}}&\tfrac{\ker d_2^{3,2}}{\operatorname{im}d_2^{1,3}}&\cdots\\
\ker d_2^{0,1}&\ker d_2^{1,1}&\tfrac{\ker d_2^{2,1}}{\operatorname{im}d_2^{0,2}}&\tfrac{\ker d_2^{3,1}}{\operatorname{im}d_2^{1,2}}&\cdots\\
E^{0,0}_2&E^{1,0}_2&\operatorname{coker}d_2^{0,1}&\operatorname{coker}d_2^{1,1}&\cdots
\end{matrix}}\right.
</math>
넷째 쪽의 처음 몇 성분들은 다음과 같다.
:<math>\left|\underline{\begin{matrix}
\vdots&\vdots&\vdots&\vdots\\
E^{0,2}_4&E^{1,2}_4&E^{2,2}_4&E^{3,2}_4&\cdots\\
E^{0,1}_4&E^{1,1}_4&E^{2,1}_4&E^{3,1}_4&\cdots\\
E^{0,0}_4&E^{1,0}_4&E^{2,0}_4&E^{3,0}_4&\cdots
\end{matrix}}\right.
\qquad=\qquad\left|\underline{\begin{matrix}
\vdots&\vdots&\vdots&\vdots\\
\ker d_3^{0,2}&\ker d_3^{1,2}&\ker d_3^{2,2}&\tfrac{\ker d^{3,2}_3}{\operatorname{im}d^{0,4}_3}&\cdots\\
\ker d_2^{0,1}&\ker d_2^{1,1}&\tfrac{\ker d_2^{2,1}}{\operatorname{im}d_2^{0,2}}&\operatorname{coker}d_3^{0,3}&\cdots\\
E^{0,0}_2&E^{1,0}_2&\operatorname{coker}d_2^{0,1}&\operatorname{coker}d_3^{0,2}&\cdots
\end{matrix}}\right.
</math>
즉, 이 스펙트럼 열은 다음과 같은 성분들로 수렴한다.
:<math>\left|\underline{\begin{matrix}
\vdots&\vdots&\vdots&\vdots\\
E^{0,2}_\infty&E^{1,2}_\infty&E^{2,2}_\infty&E^{3,2}_\infty&\cdots\\
E^{0,1}_\infty&E^{1,1}_\infty&E^{2,1}_\infty&E^{3,1}_\infty&\cdots\\
E^{0,0}_\infty&E^{1,0}_\infty&E^{2,0}_\infty&E^{3,0}_\infty&\cdots
\end{matrix}}\right.
\qquad=\qquad\left|\underline{\begin{matrix}
\vdots&\vdots&\vdots&\vdots\\
\ker d_3^{0,2}&\ker d_3^{1,2}&\ker d_3^{2,2}&\tfrac{\ker d^{3,2}_3}{\operatorname{im}d^{0,4}_3}&\cdots\\
\ker d_2^{0,1}&\ker d_2^{1,1}&\tfrac{\ker d_2^{2,1}}{\operatorname{im}d_2^{0,2}}&\operatorname{coker}d_3^{0,3}&\cdots\\
E^{0,0}_2&E^{1,0}_2&\operatorname{coker}d_2^{0,1}&\operatorname{coker}d_3^{0,2}&\cdots
\end{matrix}}\right.
</math>
수렴한 성분들이 <math>E_\infty^\bullet</math>의 [[여과 (수학)|여과]]에 의하여 주어진다고 하자.
:<math>E_\infty^{p,q}=\frac{F^pE_\infty^{p+q}}{F^{p+1}E_\infty^{p+q}}</math>
그렇다면 다음이 성립한다.
:<math>E_2^{1,0}\cong E_\infty^{1,0}=\frac{F^1E_\infty^1}{F^2E_\infty^1}=F^1E_\infty^1</math>
:<math>\ker d_2^{0,1}\cong E_\infty^{0,2}=\frac{F^0E_\infty^1}{F^1E_\infty^1}=\frac{E_\infty^1}{F^1E_\infty^1}</math>
:<math>\operatorname{coker} d_2^{0,1}\cong E_\infty^{2,0}=\frac{F^2E_\infty^2}{F^3E_\infty^2}=F^2E_\infty^2</math>
이다. 따라서, 다음과 같은 열을 적을 수 있다.
:<math>0\to E_2^{1,0}\cong F^1E_\infty^1\hookrightarrow E_\infty^1\to \frac{E_\infty^1}{F^1E_\infty^1}
\cong \ker d_2^{0,1}\hookrightarrow E_2^{0,1}\xrightarrow{d_2^{0,1}} E_2^{2,0} \twoheadrightarrow\operatorname{coker} d_2^{0,1} \cong F^2 E^2_\infty \hookrightarrow E^2_\infty</math>
여기서 <math>\ker d_2^{0,1}</math>와 <math>\operatorname{coker} d_2^{0,1}</math>을 생략하면, 5항다음과 완전열을같은 [[완전열]]을 얻는다.
:<math>0\to E_2^{1,0}\to E_\infty^1\to E_2^{0,1}\xrightarrow{d_2^{0,1}} E_2^{2,0}\to E^2_\infty</math>
이를 '''5항 [[완전열]]'''(五項完全列, {{llang|en|five-term exact sequence}})이라고 한다.
 
==== 호몰로지 5항제1 완전열사분면 스펙트럼 열 ====
마찬가지로, 수렴하는 제1 사분면 호몰로지 스펙트럼 열
:<math>E^2_{p,q}\Rightarrow_p E^\infty_{p+q}</math>
이 주어졌다고 하고, 수렴한 성분들이 <math>E^\infty__\bullet</math>의 [[여과 (수학)|여과]]에 의하여 주어진다고 하자.
이 주어졌고, <math>p<0</math> 또는 <math>q<0</math>이라면 그 성분이 자명하다고 하자. 그렇다면 다음과 같은 '''5항 [[완전열]]'''이 존재한다.
:<math>E_2E^\infty \to E_infty_{2p,0q}^2=\xrightarrowfrac{\partialF_pE^2_\infty_{2,0p+q}} E_{0,F_{p-1}E^2\to E_1^\infty \to E_infty_{1,0p+q}}^2\to 0</math>
그렇다면, 스펙트럼 열의 처음 몇 성분은 다음과 같다.
이는 구체적으로 다음과 같다. 스펙트럼 열의 성분들이 오직 제1사분면에서만 자명하지 않으므로,
:<math>E^2_{1,0}\cong E^\infty_{1,0}=\frac{F_1E_1^\infty}{F_0E_1^\infty}=\frac{E_1^\infty}{F_0E_1^\infty}</math>
:<math>\ker\partial^2_{2,0}\cong E_{2,0}^\infty =\frac{F_2E^\infty_2}{F_1E^\infty_2} =\frac{E^\infty_2}{F_1E^\infty_2}</math>
:<math>\operatorname{coker}\partial^2_{2,0}\cong E_{0,1}^\infty=\frac{F_0E_\infty^2}{F_{-1}E_\infty^2}=F_0E_\infty^2</math>
이다. 따라서이따라서, 다음과 같은 열을 적을 수 있다.
:<math>E_2^\infty \twoheadrightarrow \frac{E^\infty_2}{F_1E^\infty_2}\cong\ker\partial^2_{2,0}
\hookrightarrow E_{2,0}^2\xrightarrow{\partial^2_{2,0}} E_{0,1}^2\twoheadrightarrow\operatorname{coker}\partial^2_{2,0}\cong F_0E_1^\infty\hookrightarrow E_1^\infty \twoheadrightarrow\frac{E_1^\infty}{F_0E_1^\infty}\cong E_{1,0}^2\to 0</math>
여기서 <math>\ker\partial^2_{2,0}</math>와 <math>\operatorname{coker}\partial^2_{2,0}</math>을 생략하면, 다음과 같은 '''5항 완전열을[[완전열]]'''을 얻는다.
:<math>E_2^\infty \to E_{2,0}^2\xrightarrow{\partial^2_{2,0}} E_{0,1}^2\to E_1^\infty \to E_{1,0}^2\to 0</math>
 
== 구성 ==
줄 120 ⟶ 174:
 
== 예 ==
<math>X</math>가 [[CW 복합체]]이며, <math>X_p</math>가 그 <math>p</math>차원 뼈대라고 하자. 그렇다면 다음과 같은 완전 도형이 존재한다.<ref>{{서적 인용|이름=Raoul|성=Bott|저자고리=라울 보트|이름2=Loring W.|성2=Tu|제목=Differential forms in algebraic topology|총서=Graduate Texts in Mathematics|권=82|출판사=Springer|doi=10.1007/978-1-4757-3951-0|isbn=978-1-4419-2815-3|issn=0072-5285|언어=en}}</ref>
:<math>\begin{matrix}
&\vdots&&\vdots&&\vdots&&\vdots\\
줄 149 ⟶ 203:
* {{서적 인용|이름=John|성=McCleary|제목=History of topology|장=A history of spectral sequences: origins to 1953
|출판사=North-Holland|날짜=1999|mr=1721118|zbl=0956.55003|쪽= 631–663|doi=10.1016/B978-044482375-5/50024-9|isbn=978-0-444-82375-5|언어=en}}
* {{서적 인용|이름=Raoul|성=Bott|저자고리=라울 보트|이름2=Loring W.|성2=Tu|제목=Differential forms in algebraic topology|총서=Graduate Texts in Mathematics|권=82|출판사=Springer|doi=10.1007/978-1-4757-3951-0|isbn=978-1-4419-2815-3|issn=0072-5285|언어=en}}
 
== 바깥 고리 ==